You are on page 1of 12

Lớp: K60-A1T1

Năm học: 2017-2018


Bài Tập 1
Hạn nộp: 18/01/2018 Lí thuyết Galois

1. Cho G là một nhóm cộng. Chứng minh rằng:


(a) Với mọi số nguyên n, m và mọi x ∈ G, ta có
(nm) x = n (mx) .

(b) Nếu G là nhóm giao hoán thì với mọi số nguyên n và mọi x, y ∈ G, ta có
n (x + y) = nx + ny.

Chứng minh:
(a) Với mọi số nguyên n, m và mọi x ∈ G, ta có

{z· · · + x}
(nm) x = |x + x +
nm lần
=x {z· · · + x} + x
| +x+ {z· · · + x} + · · · + x
| +x+ {z· · · + x} = n (mx) .
| +x+
m lần m lần m lần
| {z }
n lần

(b) Nếu G là nhóm giao hoán thì với mọi số nguyên n và mọi x, y ∈ G, ta có

n (x + y) = (x + y) + (x + y) + · · · + (x + y)
| {z }
n lần
=x {z· · · + x} + y| + y +
| +x+ {z
··· + y
}
n lần n lần
= nx + ny.

2. Phát biểu và chứng minh các kết quả tương tự cho nhóm nhân.

Chứng minh:
(a) Với mọi số nguyên n, m và mọi x ∈ G, ta có
xnm = (xm )n .

(b) Nếu G là nhóm giao hoán thì với mọi số nguyên n và mọi x, y ∈ G, ta có
(xy)n = xn y n .
K60-A1T1 Bài Tập 1 - Trang 2 18/01/2018

3. Chứng minh rằng:


(a) Tập Bij (S) các song ánh từ một tập hợp S vào chính nó cùng với phép hợp thành
là một nhóm.
(b) Tập GL (V ) các phép tự đẳng cấu của một không gian véc-tơ thực V cùng với phép
hợp thành là một nhóm.
(c) Tập GLn (R) các ma trận vuông thực khả nghịch cấp n cùng với phép nhân ma
trận tạo thành một nhóm.
(d) Tập hợp SLn (R) các ma trận vuông thực cấp n có định thức bằng 1 là một nhóm
con của GLn (R).

Chứng minh:
(a) Tập Bij (S) các song ánh từ một tập hợp S vào chính nó cùng với phép hợp
thành là một nhóm. Phần tử trung lập của nhóm Bij (S) là ánh xạ đồng nhất
IdS . Mỗi ánh xạ f ∈ Bij (S) là một song ánh nên có ánh xạ ngược f −1 cũng là
một song ánh. Đây chính là phần tử nghịch đảo của f .

(b) Tập GL (V ) các phép tự đẳng cấu của một không gian véc-tơ thực V cùng với
phép hợp thành là một nhóm con của nhóm Bij (V ).

(c) Tập GLn (R) các ma trận vuông thực khả nghịch cấp n cùng với phép nhân ma
trận tạo thành một nhóm. Phần tử đơn vị của nhóm này chính là ma trận đơn
vị In . Mỗi ma trận M ∈ GLn (R) có ma trận khả nghịch M −1 thỏa mãn

M M −1 = In = M −1 M.

Đây chính là phần tử nghịch đảo của M .

(d) Tập hợp SLn (R) các ma trận vuông thực cấp n có định thức bằng 1 là một
nhóm con của GLn (R). Thật vậy, dễ dàng kiểm tra rằng SLn (R) chính là một
nhóm con của GLn (R) và do đó cũng là một nhóm.

4. Cho G là một nhóm và Hi , i ∈ I là một họ các nhóm con của G. Chứng minh rằng giao
của các Hi , i ∈ I cũng là một nhóm con của G.

Chứng minh:
Kí hiệu H là giao của các Hi , i ∈ I. Nhận xét rằng do mỗi Hi là một nhóm nên ta
có e ∈ Hi với mọi i ∈ I. Vì vậy, ta suy ra e ∈ H. Giả sử x, y ∈ H. Khi đó, ta có
x, y ∈ Hi với mọi i ∈ I. Vì vậy, ta có xy ∈ Hi và x−1 ∈ Hi với mọi i ∈ I. Vì thế, ta
có xy ∈ H và x−1 ∈ H. Nói cách khác, H là một nhóm con của G.
K60-A1T1 Bài Tập 1 - Trang 3 18/01/2018

5. Cho G là một nhóm và A là một tập con của G. Chứng minh rằng hAi là một nhóm con
của G.

Chứng minh:
Thật vậy, từ định nghĩa của hAi, ta có hAi là giao của tất cả các nhóm con của G
chứa A. Theo Bài tập 4, ta có hAi là một nhóm con của G.

6. Lựa chọn các ý đúng trong các ý sau:


Nhóm GL (R5 ) là một nhóm giao hoán.
Nhóm GL4 (R) là một nhóm không giao hoán.
Nhóm Z là một nhóm giao hoán.
Nhóm Q× là một nhóm không giao hoán.

Chứng minh:
Các ý sau đây là đúng:

• Nhóm GL4 (R) là một nhóm không giao hoán.

• Nhóm Z là một nhóm giao hoán.

7. Cho G là một nhóm. Chứng minh rằng:


(a) Nếu x ∈ G là một phần tử thỏa mãn xg = g = gx với mọi g ∈ G thì x = e.
(b) Nếu x, y, z là các phần tử của G sao cho xy = xz thì y = z.
(c) Nếu x, y, z là các phần tử của G sao cho yx = zx thì y = z.

Chứng minh:
(a) Thật vậy, ta có

xe = e = ee.

Theo quy tắc giản ước ta suy ra x = e.

(b) Nhân cả hai vế của đẳng thức về bên trái với x−1 ta suy ra điều phải chứng
minh.

(c) Nhân cả hai vế của đẳng thức về bên phải với x−1 ta suy ra điều phải chứng
minh.
K60-A1T1 Bài Tập 1 - Trang 4 18/01/2018

8. Cho G là một nhóm và H là một nhóm con của G. Chứng minh rằng với mọi x ∈ G thì
xHx−1 là một nhóm con của G.

Chứng minh:
Nhận xét rằng

xHx−1 = xhx−1 h ∈ H .


Do đó, ta có e = xex−1 ∈ xHx−1 . Với mọi h, g ∈ H, ta có


−1
xhx−1 = xh−1 x−1 ∈ xHx−1
xhx−1 xgx−1 = xhgx−1 ∈ xHx−1 .
 

Vì vậy xHx−1 là một nhóm con của G.

9. Chứng minh rằng nếu f : G → H là một đồng cấu giữa hai nhóm G, H thì

• f (eG ) = eH ; • f (x−1 ) = (f (x))−1 , ∀x ∈ G.

Ở đây, eG , eH tương ứng là các phần tử trung lập của G và H.

Chứng minh:
Ta có

f (eG ) eH = f (eG ) = f (eG eG ) = f (eG ) f (eG ) .

Theo luật giản ước ta có f (eG ) = eH . Hơn nữa, ta có:

f x−1 f (x) = f x−1 x = f (eG ) = eH .


 

Tương tự, ta cũng có

f (x) f x−1 = eH .


Vì vậy, ta có điều phải chứng minh

f x−1 = (f (x))−1 , ∀x ∈ G.


10. Chứng minh rằng nếu G là một nhóm hữu hạn và H là một nhóm con của G thì ta có

|G| = [G : H] |H| .
K60-A1T1 Bài Tập 1 - Trang 5 18/01/2018

Chứng minh:
Cho x, y ∈ G. Giả sử xH ∩ yH 6= ∅. Ta sẽ chứng minh xH = yH. Thật vậy, giả
sử z ∈ xH ∩ yH. Khi đó, tồn tại a, b ∈ H sao cho xa = yb = z. Do đó, ta có
x−1 y = ab−1 ∈ H. Ta sẽ chứng minh yH ⊂ xH. Thật vậy, giả sử c ∈ yH. Gọi d ∈ H
sao cho c = yd. Khi đó, ta có c = x (x−1 y) d. Do (x−1 y) d ∈ H nên ta có c ∈ xH. Vì
vậy, ta suy ra yH ⊂ xH. Chứng minh tương tự, ta có xH ⊂ yH. Mặt khác, ánh xạ
f : xH → yH, xa 7→ ya và g : yH → xH, ya 7→ xa là hai ánh xạ ngược của nhau. Do
đó, f là một song ánh. Nói cách khác, với mọi x, y ∈ G, tập hợp xH và yH có cùng
lực lượng. Hơn nữa, ta có
[
G= xH.
x∈G

Do vậy

|G| = [G : H] |H| .

Chứng minh:
Trước tiên, ta chứng minh rằng Z (G) là một nhóm con của G. Thật vậy, dễ thấy
e ∈ Z (G). Giả sử x, y ∈ Z (G). Khi đó, với mọi g ∈ G ta có:

(xy) g = x (yg) = x (gy) = (xg) y = (gx) y = g (xy)


−1 −1
x−1 g = g −1 x = xg −1 = gx−1 .

Vậy, ta có Z (G) là một nhóm con của G. Mặt khác, với mọi a, b ∈ G và c ∈ Z (G),
ta có

aca−1 b = caa−1 b = cb = b.
 

Vì thế, aca−1 ∈ Z (G). Vậy, ta suy ra aZ (G) a−1 ⊂ Z (G) với mọi a ∈ G. Nói cách
khác, a−1 Z (G) a ⊂ Z (G) với mọi a ∈ G. Từ đây, ta suy ra aZ (G) a−1 ⊂ Z (G) với
mọi a ∈ G. Vậy, ta có đẳng thức

aZ (G) a−1 = Z (G)

với mọi a ∈ G. Ta có điều phải chứng minh.


K60-A1T1 Bài Tập 1 - Trang 6 18/01/2018

11. Chứng minh rằng:


(a) Mọi nhóm con của nhóm giao hoán đều là nhóm con chuẩn tắc.
(b) Cho một nhóm G, ta định nghĩa tâm của G là

Z (G) = {x ∈ G |xg = gx, ∀g ∈ G} .

Khi đó, Z (G) và mọi nhóm con của nó đều là nhóm con chuẩn tắc của G.
(c) Cho G = S3 và H = he, (1, 2)i. Khi đó H không phải là một nhóm con chuẩn tắc
của G.

Chứng minh:
(a) Giả sử G là một nhóm giao hoán và H là một nhóm con bất kì của G. Khi đó,
với mọi x ∈ G, do phép toán của G có tính giao hoán nên xH = Hx. Vì thế
xHx−1 = H. Nói cách khác, H là một nhóm con chuẩn tắc.

(b) Giả sử H là một nhóm con của Z (G). Cho x là một phần tử tùy ý của G. Theo
định nghĩa, ta có xh = hx với mọi h ∈ H. Vì thế, xHx−1 = H. Nói cách khác,
mọi nhóm con của Z (G) đều là nhóm con chuẩn tắc của G.

(c) Thật vậy, chọn g = (1, 3). Khi đó ta có

gH = {g, (1, 3) (1, 2)} = {(1, 3) , (1, 2, 3)} ,


Hg = {g, (1, 2) (1, 3)} = {(1, 3) , (1, 3, 2)} .

Do đó gH 6= Hg.

12. Chứng minh rằng hạt nhân của một đồng cấu nhóm f : G → H là một nhóm con chuẩn
tắc của G.

Chứng minh:
Thật vậy, giả sử x ∈ G và y ∈ Ker (f ). Khi đó, ta có

f xyx−1 = f (x) f (y) f x−1 = f (x) f x−1 = eH .


  

Do đó, ta có xyx−1 ∈ Ker (f ).

13. Giả sử H là một nhóm con của nhóm G. Chứng minh rằng các khẳng định sau là tương
đương:
(a) Nhóm H là nhóm con chuẩn tắc.
(b) Ta có xHx−1 = H với mọi x ∈ G.
K60-A1T1 Bài Tập 1 - Trang 7 18/01/2018

(c) Ta có xHyH = xyH với mọi x, y ∈ G.


(d) Ánh xạ
G/H × G/H → G/H
([x] , [y]) 7→ [xy]
cùng với phần tử [e] và ánh xạ
i : G/H → G/H,
[x] 7→ x−1
 

biến G/H thành một nhóm. Ta gọi G/H là nhóm thương của G bởi nhóm con
chuẩn tắc H.

Chứng minh:

• (a) ⇔ (b): Ta có

xH = Hx ⇔ (xH) x−1 = (Hx) x−1 ⇔ xHx−1 = H.

• (a) ⇔ (c): Chú ý rằng, nếu yH = Hy thì

xHyH = xyHH = xyH.

Ngược lại, chọn y = x−1 ta có

xHx−1 H = xx−1 H = H ⇒ xHx−1 = H ⇒ xH = Hx.

• (c) ⇒ (d): Các tính chất của nhóm có thể dễ dàng được kiểm tra. Ta chỉ cần
kiểm tra tính đúng đắn của các ánh xạ đã cho. Nếu [x] = [a] và [y] = [b] thì

[xy] = [ab]
 −1   −1 
x = a .

Thật vậy, theo giả thiết ta có x−1 a, y −1 b ∈ H. Do đó, ta có

(xy)−1 ab, xa−1 ∈ H

là điều phải chứng minh.

• (d) ⇒ (b): Ta chỉ cần chứng minh xhx−1 ∈ H với mọi x ∈ G. Thật vậy, ta có
[x] = [xh]. Do đó, ta có

xhx−1 = [xh] x−1 = [x] x−1 = [e] .


     

Nói cách khác, xhx−1 ∈ H.


K60-A1T1 Bài Tập 1 - Trang 8 18/01/2018

14. Chứng minh rằng, nếu H là một nhóm con chỉ số 2 của nhóm G thì H là một nhóm con
chuẩn tắc của G.

Chứng minh:
Do chỉ số của H trong G là 2 nên tồn tại x ∈
/ H sao cho G = H ∪ xH. Vì H 6= xH
nên ta suy ra H 6= Hx vì ngược lại ta có x ∈ H. Vậy, ta suy ra H ∩ Hx = ∅. Nói
cách khác, ta có G = H ∪ Hx. Vì vậy, Hx = xH và H là nhóm con chuẩn tắc của G.

15. Chứng minh rằng quan hệ đẳng cấu của hai nhóm là một quan hệ tương đương trên tập
hợp các nhóm.

Chứng minh:
Dễ thấy phép đồng nhất G → G là một đẳng cấu nhóm. Do đó quan hệ đẳng cấu có
tính phản xạ. Nếu f là một đẳng cấu thì ánh xạ ngược f −1 cũng là một đẳng cấu.
Do đó quan hệ đẳng cấu có tính đối xứng. Cuối cùng, vì hợp thành của hai đẳng cấu
là một đẳng cấu nên ta có tính bắc cầu của quan hệ đẳng cấu. Nói cách khác, quan
hệ đẳng cấu của hai nhóm là một quan hệ tương đương trên tập hợp các nhóm.

16. Cho f : G → H là một đồng cấu nhóm. Chứng minh rằng, hai nhóm Im (f ) và G/Ker (f )
là đẳng cấu.

Chứng minh:
Dễ dàng kiểm tra rằng

f¯ : G/Ker (f ) → H
[x] 7→ f (x)

được định nghĩa tốt và là một đồng cấu nhóm. Nó cảm sinh một toàn cấu nhóm

g : G/Ker (f ) → Im (f )
[x] 7→ f (x) .

Nếu [x] ∈ Ker (g) khi và chỉ khi f (x) = 0. Do đó Ker (g) = {[e]}.Vậy, g là một đẳng
cấu.

17. Chứng minh rằng nhóm thương GLn (Q) /SLn (Q) đẳng cấu với nhóm nhân Q× .
K60-A1T1 Bài Tập 1 - Trang 9 18/01/2018

Chứng minh:
Đồng cấu det : Gln (Q) → Q \ {0} là một toàn cấu nhóm với hạt nhân Sln (Q). Ta
có đẳng cấu

Gln (Q) /Sln (Q) ∼


= Q \ {0} .

18. Cho S là một tập hợp trên đó có tác động của một nhóm G. Chứng minh rằng, với mọi
s ∈ S, nhóm con ổn định Gs là một nhóm con của G.

Chứng minh:
Thật vậy, dễ thấy e ∈ Gs . Hơn nữa, nếu x, y ∈ Gs thì ta có:

(xy) s = x (ys) = xs = s
x−1 s = x−1 (xs) = s.

Vậy xy, x−1 ∈ Gs hay Gs là một nhóm con của G.

19. Nhắc lại rằng An là nhóm con của nhóm các phép thế cấp n có dấu bằng 1. Chứng minh
rằng:
(a) Nhóm A3 là một nhóm đơn sinh.
(b) Kết luận rằng S3 là một nhóm giải được.
(c) Tập hợp

K = {e, (1, 2) (3, 4) , (1, 3) (2, 4) , (1, 4) (2, 3)}

là một nhóm con chuẩn tắc của A4 .


(d) Nhóm thương A4 /K là một nhóm giao hoán.
(e) Kết luận rằng S4 là một nhóm giải được.

Chứng minh:
(a) Ta có |A3 | là một số nguyên tố. Do đó, nhóm A3 là một nhóm đơn sinh.

(b) Nhắc lại rằng ánh xạ

sgn : S3 → {±1}
σ 7→ sgn (σ)
K60-A1T1 Bài Tập 1 - Trang 10 18/01/2018

là một toàn cấu nhóm với hạt nhân A3 . Do vậy, S3 /A3 ∼


= {±1} là một nhóm
giao hoán. Mặt khác, A3 là một nhóm giao hoán nên theo định nghĩa S3 là một
nhóm giải được.

(c) Ta cần chứng minh σK = Kσ với mọi σ ∈ A4 . Tuy nhiên, do mọi phần tử của
A4 đều là tích của một số chẵn các phép thế sơ cấp, ta chỉ cần chứng minh điều
này với các tích của hai phép thế sơ cấp bất kì.

(d) Do [A4 : K] = 3 nên A4 /K là nhóm giao hoán.

(e) Tương tự Ý (b).

20. Cho f : G → H là một đồng cấu nhóm.


(a) Chứng minh rằng f (DG) ⊂ DH.
(b) Kết luận rằng ánh xạ cảm sinh f |DG là một đồng cấu nhóm DG → DH. Ta ký
hiệu ánh xạ này là D (f ).
(c) Chứng minh rằng nếu f là đơn ánh thì D (f ) cũng là đơn ánh.
(d) Chứng minh rằng nếu f là toàn ánh thì D (f ) cũng là toàn ánh.

Chứng minh:
(a) Nhắc lại rằng, nếu X là một tập con của G và T là một nhóm con của H sao cho
f (X) ⊂ T thì f (hXi) ⊂ T . Vậy, ta chỉ cần chứng minh f ([x, y]) ∈ DH với mọi
x, y ∈ G. Tuy nhiên, điều này là hệ quả của đẳng thức f ([x, y]) = [f (x) , f (y)].

(b) Như vậy, hạn chế của đồng cấu f trên DG cảm sinh một ánh xạ

D (f ) : DG → DH, x 7→ f (x) .

Dễ thấy D (f ) là một đồng cấu nhóm do f là đồng cấu nhóm.

(c) Dễ thấy Ker (D (f )) ⊂ Ker (f ). Vì vậy, nếu f là đơn ánh thì D (f ) cũng vậy.

(d) Nhắc lại rằng nếu g : T → S là một đồng cấu nhóm và Y là một hệ sinh
của S sao cho Y ⊂ Im (g) thì g là toàn cấu. Như vậy, ta chỉ cần kiểm tra
[x, y] ∈ Im (D (f )) với mọi x, y ∈ H. Thật vậy, do f là một toàn cấu nên tồn
tại a, b ∈ G sao cho f (a) = x và f (b) = y. Khi đó ta có [x, y] = [f (a) , f (b)].

21. Cho H là một nhóm con chuẩn tắc của nhóm G. Chứng minh rằng nhóm G là một nhóm
giải được nếu và chỉ nếu H và G/H là các nhóm giải được.
K60-A1T1 Bài Tập 1 - Trang 11 18/01/2018

Chứng minh:
Nếu G là một nhóm giải được thì tồn tại một số tự nhiên n sao cho Dn G = {e}. Do
Dn H ⊂ Dn G nên Dn H = {e}. Mặt khác, ánh xạ cảm sinh Dn G → Dn (G/H) là
một toàn cấu nên Dn (G/H) = {e}. Như vậy, H và G/H là các nhóm giải được.
Giả sử H và G/H là các nhóm giải được, ta sẽ chứng minh G cũng là một nhóm giải
được. Thật vậy, giả sử n, m là các số tự nhiên sao cho Dn H = {e} và Dm (G/H) =
{e}. Nhận thấy đồng cấu chiếu p : G → G/H cảm sinh một đồng cấu nhóm

p̄ : Dm G → Dm (G/H)
x 7→ p (x) .

Vậy, nếu x ∈ Dm G thì p (x) = e. Nói cách khác, x ∈ Ker (p) = H hay Dm G ⊂ H.
Vậy, ta kết luận Dn+m G = {e} hay G là một nhóm giải được.

22. Cho n ≥ 3 là một số tự nhiên.


(a) Chứng minh rằng, với bộ số tự nhiên phân biệt i, j, k ≤ n, ta có đẳng thức sau đây
trong Sn :

(i, j, k) = (i, k) (i, j) .

(b) Chứng minh rằng, nếu i, j, k, l ≤ n là các số tự nhiên phân biệt thì

(i, j, k) (j, k, l) = (i, j) (k, l) .

(c) Chứng minh rằng An được sinh bởi các xích có độ dài 3.

Chứng minh:
(a) Hiển nhiên.

(b) Hiển nhiên.

(c) Nhắc lại rằng một phần tử của An là tích của một số chẵn các phép thế sơ cấp.
Tuy nhiên, ta thấy rằng tích của hai phép thế sơ cấp bất kì có thể biểu diễn
qua các xích có độ dài 3. Vậy, An được sinh bởi các xích có độ dài 3.

23. Cho n ≥ 5 là một số tự nhiên. Cho (a, b, c) là một xích có độ dài 3 của Sn .
(a) Chứng minh rằng tồn tại σ ∈ Sn sao cho

σ (a, b, c) σ −1 = (a, c, b) = (a, b, c) (a, b, c) .


K60-A1T1 Bài Tập 1 - Trang 12 18/01/2018

(b) Giả sử d, e ≤ n là hai số tự nhiên sao cho a, b, c, d, e là các số đôi một phân biệt.
Chứng minh rằng

σ (d, e) (a, b, c) (d, e) σ −1 = (a, c, b) .

(c) Kết luận rằng tồn tại α ∈ An sao cho

(a, b, c) = α (a, b, c) α−1 (a, b, c)−1 .

(d) Chứng minh rằng D (An ) = An .


(e) Chứng minh rằng Sn là nhóm không giải được.

Chứng minh:
(a) Chọn σ = (b, c). Ta có:

(b, c) (a, b, c) (b, c) (a) = c,


(b, c) (a, b, c) (b, c) (b) = a,
(b, c) (a, b, c) (b, c) (c) = b.

Vậy, ta có

(b, c) (a, b, c) (b, c) = (a, c, b) .

(b) Nếu d, e ≤ n là hai số tự nhiên sao cho a, b, c, d, e là các số đôi một phân biệt
thì ta có

σ (d, e) (a, b, c) (d, e) σ −1 = σ (a, b, c) (d, e) (d, e) σ −1 = (a, c, b) .

(c) Từ hai ý trên, ta suy ra luôn tồn tại α ∈ An sao cho

α (a, b, c) α−1 = (a, c, b) = (a, b, c) (a, b, c) .

Vậy, ta có

(a, b, c) = α (a, b, c) α−1 (a, b, c)−1 ∈ D (An ) .

(d) Như vậy, ta kết luận mọi xích có độ dài 3 đều thuộc D (An ). Do các xích có độ
dài 3 sinh ra An nên ta có D (An ) = An .

(e) Ta kết luận An không phải là nhóm giải được do với mọi số tự nhiên k ta có
Dk (An ) = An . Mặt khác, do An là một nhóm con chuẩn tắc của Sn nên Sn
không phải là nhóm giải được.

You might also like